Đến nội dung

Bui Ba Anh

Bui Ba Anh

Đăng ký: 02-05-2014
Offline Đăng nhập: Riêng tư
***--

#644679 Thảo luận về Đề thi và Lời giải của IMO 2016

Gửi bởi Bui Ba Anh trong 12-07-2016 - 16:12

Bài 4: Ta gọi tập thỏa mãn đề bài là tập chuẩn

Giả sử với số $b$ nào đó ta thu được tập chuẩn $X$, ta sẽ tìm giá trị nhỏ nhất của $b$

Với mỗi phần tử trong  $X$, nếu nó có ước nguyên tố chung với phần tử nào đó trong các phần tử còn lại thì ta xếp các ước nguyên tố đó vào tập $P$. Sau quá trình này, nếu có nhiều phần tử trùng nhau trong $P$ thì ta chọn một trong chúng

Ta sẽ xét các trường hợp

*TH1: Nếu trong $P$ tồn tại các số nguyên tố lớn hơn hoặc bằng $11$, gọi $p$ là một số như vậy

Khi đó tồn tại $P(a+m)$ và $P(a+n)$ chia hết cho $p$ ($m>n$)

Suy ra $$p|P(a+m)-P(a+n)$$

$$=>p|(m-n)(m+n+1)$$

-Nếu $p|m-n$, khi đó do $m$ khác $n$ nên $m \geq 12$ hay $b \geq 12$

-Nếu $p|m+n+1$ khi đó do $m$ khác $n$ nên $m \geq 6$ hay $ b\geq 6$

Vậy trong trường hợp này $b \geq 6$

*TH2: Nếu trong $P$ chỉ chứa các số nguyên tố $3,5,7$ (vì các phân tử đều là số lẻ)

Rõ ràng $P$ không chưa $5$ vì $n^2+n+1$ không chia hết cho $5$

Ta có $2$ nhận xét:

1. $n^2+n+1$ chia hết cho $3$ khi và chỉ khi $n=1$ mod 3, cũng suy ra là trong ba số liên tiếp thì chỉ có một số $t$ mà $P(t)$ chia hết cho $3$

2.$n^2+n+1$ chia hết cho $7$ khi và chỉ khi $n=2,4$ mod 7, cũng suy ra là trong ba số liên tiếp thì có nhiều nhất hai số mà $P$ của nó chia hết cho $7$,trong 2 số liên tiếp thì có nhiều nhất một số mà $P$ của nó chia hết cho $7$

Trở lại trường hợp này

-Nếu $X$ có $2$ phần tử, điều này vô lí vì số phần tử chia hết cho $3$ là $0$ hoặc $2$, nếu là $0$ thì hai số đó đều chia hết cho $7$ vô lí theo nhận xét. nếu có $2$ số chia hết cho $3$ cũng vô lí theo nhận xét

-Nếu $X$ có $3$ phần tử, nếu có $0$ phần tử chia hết cho $3$, suy ra tất cả đều chia hết cho $7$ là vô lí theo nx2. Nếu có $2,3$ số chia hết cho $3$ vô lí theo nx1.

-Nếu $X$ có $4$ phần tử, nếu có $0$ phần tử chia hết cho $3$ là vô lí theo nx1, nếu có nhiều hơn $2$ phần tử chia hết cho $3$ thì chỉ có thể là $P(a+1),P(a+4)$, suy ra $P(a+2),P(a+3)$ chia hết cho $7$ vô lí theo nx2

-Nếu có $5$ phần tử: Có $0$ phần tử chia hết cho $3$ là vô lí, nếu có $2$ phần tử chia hết cho $3$, dễ thấy có $2$ phần tử liên tiếp trong $X$ chia hết cho $7$ vô lí theo nx2.

Tóm lại trong trường hợp này $b \geq 6$

Ta sẽ chỉ ra min $b$ là 6

Xét số $n$ thỏa mãn: n=0 mod 3, n=5 mod 11, n=6 mod 7. Số này là tồn tại theo định lí số dư trung hoa

Khi đó $3 \mid P(x+1), P(x+4),19 \mid P(x+2),P(x+6),  7 \mid P(x+3), 7 \mid P(x+5)$.




#644508 Thảo luận về Đề thi và Lời giải của IMO 2016

Gửi bởi Bui Ba Anh trong 11-07-2016 - 15:34

Bài $1$ khá hay. Gọi $CD$ cắt $BF$ tại $T$ và dễ dàng chứng minh $E$ là trung điểm $FT$ (Chú ý cách xác định điểm $E$ là giao của trung trực $AD$ và đường thẳng đổi xứng của $AC$ qua $AD$)

Khi đó tứ giác $XMFE$ nội tiếp, ngũ giác $BMDEA$ và $BFDXC$ nội tiếp.

Suy ra $EM,FX,BD$ đồng quy tại tâm đẳng phương của ba đường tròn.




#636602 VMF's Marathon Hình học Olympic

Gửi bởi Bui Ba Anh trong 29-05-2016 - 19:48

Mình xin lỗi vì quên đăng bài đề nghị

$\boxed{\text{Bài toán 15.}}$  Đường tròn $W_1$ và $W_2$ giao nhau tại $P,K$. $XY$ là tiếp tuyến chung ngoài gần $P$ hơn của $W_1,W_2$ với $X$ thuộc $W_1$ và $Y$ thuộc $W_2$. $XP$ cắt $W_2$ tại điểm thứ hai $C$ và $YP$ cắt $W_1$ tại điểm thứ hai $B$. Gọi $A$ là giao của $BX,CY$. Chứng minh rằng nếu $Q$ là giao điểm còn lại của $(ABC)$ và $(AXY)$ thì góc $QXA$ bằng góc $QKP$

Nguồn: sưu tầm

 

 




#636385 VMF's Marathon Hình học Olympic

Gửi bởi Bui Ba Anh trong 28-05-2016 - 22:17

$\boxed{\text{Giải bài 14.}}$ Gọi $D,E,F$ là tiếp điểm của $(I)$ với $BC,AC,AB$, giả sử $B_1,C_2$ thuộc $BC$,$B_2,A_1$ thuộc $AB$, $A_2,C_1$ thuộc $AC$, gọi các đường tròn nhỏ là $(u),(v),(w)$ thứ tự ứng với đỉnh $A,B,C$

Ta sẽ chứng minh $XD,YE,ZF$ đồng quy tại $P$

Thật vậy dễ thấy các tam giác $B_1XC_2,B_2A_1Z,YA_2C_1$ nội tiếp. (1)

Mà $\dfrac{sin(A_1ZF)}{sin(FZB_1)}=\dfrac{A_1F.ZB_2}{B_2F.ZA_1}$, thiết lập các đẳng thức tương tự và chú ý (1), áp dụng Ceva dạng sin ta có đpcm

Mặt khác tam giác $DEF$ đồng dạng $XYZ$ và $XD,YE,ZF$ đồng quy tại $P$ nên tồn tại phép vị tự tâm $P$ biến $DEF$ thành $XYZ$, do đó biến $I$ thành $M$ nên $P,I,M$ thẳng hàng.

Giả sử tiếp tuyến chung trong của $(v)$ và $(I)$ cắt $BD,XD,FZ$ tại $K,L,R$ và tiếp tuyến chung trong của $(w)$ và $(I)$ cắt $CD,XD,EY$ tại $H,L',Q$

Do $B_1K=KD$ và $DH=C_2H$ nên $DL=LX$ và $DL'=L'X$ theo talet hay các tiếp tuyến này cắt nhau tại một điểm trên $DX$

Chứng minh tương tự cho các tiếp tuyến chung còn lại

Khi đó tam giác $LQR$ đồng dạng $XYZ$ và $XL,YQ,CR$ đồng quy tại $P$ nên tồn tại phép vị tự biến $LQR$ thành $XYZ$ và do đó biến $I$ thành $N$ hay $P,I,N$ thẳng hàng

Vậy ta có đpcm.

$$\begin{array}{| l | l |} \hline Ngockhanh99k48 & 1\\ \hline IHateMath & 1\\ \hline fatcat12345 & 2\\ \hline dogsteven & 3\\ \hline baopbc & 4\\ \hline QuangDuong12011998 & 1\\ \hline xuantrandong & 1\\ \hline mrjackass & 1\\ \hline vietnaminmyheart & 1\\ \hline BuiBaAnh & 1\\ \hline\end{array}$$




#632008 ĐỀ THI OLYMPIC CHUYÊN KHOA HỌC TỰ NHIÊN 2016

Gửi bởi Bui Ba Anh trong 08-05-2016 - 21:23

 

Câu 7. Cho $a, b, c$ là các số thực dương thỏa mãn $ab + bc + ca + 2abc = 1$. Chứng minh rằng $$\sum\frac{a(a + 1)}{(2a + 1)^{2}}\le \frac{9}{16}$$

 

 

 

Tiếp cận bằng dồn biến

Đặt $f(a,b,c)=\dfrac{a(a+1)}{(2a+1)^2}+\dfrac{b(b+1)}{(2b+1)^2}+\dfrac{c(c+1)}{(2c+1)^2}$

Ta sẽ chứng minh $f(a,t,t) \leq \dfrac{9}{16}$ với $t$ thỏa mãn $2at^2+2at+t^2=1$

Tức là chứng minh $\dfrac{a(a+1)}{(2a+1)^2}+2\dfrac{t(t+1)}{(2t+1)^2} \leq \dfrac{9}{16}$

$$<=> f(t)=\dfrac{1-t^2}{4}+2\dfrac{t(t+1)}{(2t+1)^2} \leq \dfrac{9}{16}$$

Xét $f'(t)=\dfrac{-t}{2}+\dfrac{2}{(2t+1)^3}=0<=>(2t-1)(4t^3+8t^2+7t+4) =0<=>t=0,5$

Dùng bảng biến thiên dễ suy ra $f(t) \leq f(0,5)=\dfrac{9}{16}$

Tiếp theo ta sẽ chứng minh $f(a,b,c) \leq f(a,t,t)$ với $t =\sqrt{bc}(1)$

Khi đó nếu $(1)$ đúng thì ta có thể đưa về trường hợp trên và ta có đpcm

$$f(a,b,c) \leq f(a,t,t)$$

$$<=>\dfrac{b(b+1)}{(2b+1)^2}+\dfrac{c(c+1)}{(2c+1)^2} \leq \dfrac{2\sqrt{bc}(\sqrt{bc}+1)}{(2\sqrt{bc}+1)^2}$$

$$<=>(y-z)^2.\dfrac{-y^3z^3+(y^2+yz+z^2)-4y^2z^2(y^2+yz+z^2)-4yz(y^2+yz+z^2)-3yz+1}{(2y^2+1)^2(2z+1)^2(2yz+1)^2} \leq 0(b=y^2,c=z^2)(2)$$

Đặt $m=y^2+yz+z^2$,$n=yz$,$m \geq 3n$ 

$$(2)<=>m(1-4n^2-4n)-4n^3-2n+1 \leq0$$

Do $2abc+ab+ac+bc=1$ nên không mất tổng quát ta có thể giả sử $bc \geq \dfrac{1}{4}=>n \geq \dfrac{1}{2}=>1-4n^2-4n<0$

Suy ra $m(1-4n^2-4n)-4n^3-2n+1 \leq -16n^3-12n^2+n+1=g(n) \leq 0$ với $1 >n \geq \dfrac{1}{2}$. Điều này đúng do $g(n)$ nghịch biến trên tập xác định




#628386 xác định số $n$ bé nhất sao cho từ các điều kiện cho trước

Gửi bởi Bui Ba Anh trong 19-04-2016 - 23:27

Biểu thị mỗi học sinh là một điểm trên mặt phẳng sao cho không có ba điểm nào thẳng hàng.

Cứ $2$ học sinh đạt điểm tối ưu cho cùng một môn được nối với nhau bởi 1 đoạn thẳng xác định bởi 1 màu nào đó

Do điều kiện của bài toán cứ $3$ học sinh đạt điểm tối ưu cho một môn sẽ biểu thị bằng một tam giác có $3$ cạnh cùng màu, và giữa $2$ tam giác bất kì có đúng $1$ điểm chung

Nhận xét 1: Nếu $4$ tam giác, với mỗi tam giác chỉ có các cạnh cùng màu, có chung $1$ đỉnh, thì tất cả các tam giác khác sẽ cùng có chung đỉnh này.

Thật vậy. Xét $4$ tam giác này, giả sử tam giác thứ $5$ có đỉnh không là đỉnh chung của $4$ tam giác này, do điều kiện $2$ tam giác bất kì có $1$ đỉnh chung nên theo $\Dirichle$ tồn tại $2$ tam giác trong $4$ tam giác đó có chung đỉnh với tam giác thứ $5$. Suy ra $2$ tam giác này có $2$ đỉnh chung, vô lí. Nên điều giả sử sai và ta có đpcm

Nhận xét 2: Điều kiện $n=4$ là điều kiện bé nhất để tất cả các tam giác chung đỉnh.( $n \geq 3$ nên dễ dàng tìm phản ví dụ cho $3$)

Trở lại bài toán

-Với $n$ tam giác, xét một tam giác bất kì, từ nhận xét, suy ra phải tồn tại $1$ đỉnh của nó chung đỉnh của $3$ tam giác khác

-Theo $\Dirichle$ suy ra số tam giác bé nhất để đảm bảo luôn có $3$ tam giác chung đỉnh với tam giác đang xét là:$3.2+1=7$ tam giác. Cộng với tam giác đang xét. Suy ra $n=8$




#609567 $2\sum \dfrac{AA_2}{AA_1} \geq 1+\dfrac{4r}{R}$

Gửi bởi Bui Ba Anh trong 18-01-2016 - 01:12

Cho tam giác $ABC$ nội tiếp đường tròn $(O)$ bán kính $R$, ngoại tiếp đường tròn $(I)$ bán kính $r$. Các đường trung tuyến $AA_1,BB_1,CC_1$

Tiếp tuyến tại $B,C$ của đường tròn $(O)$ cắt nhau tại $S$, $AS$ cắt $BC$ tại $A_2$. Các điểm $B_2,C_2$ xác định tương tự

Chứng minh rằng:

$\dfrac{AA_2}{AA_1}+\dfrac{BB_2}{BB_1}+\dfrac{CC_2}{CC_1} \geq \dfrac{1}{2}+\dfrac{2r}{R}$




#608021 $a-b|a$ và $a-b|b$

Gửi bởi Bui Ba Anh trong 08-01-2016 - 20:19

Ta chứng minh bằng qui nạp. Giả sử bài toán đúng tới $n$ tức là tồn tại tập $S_{n}=\left \{ a_1,a_2,...a_{n} \right \}$  thỏa đề

Ta xây dựng tập $S_{n+1}$ như sau : Đặt $A=\prod (a_i-a_j)$ với $i,j$ chạy từ $1$ đến $n$

Với mỗi $k$ từ $1$ đến $n$ ta chọn $b_k=Aa_1a_2...a_n+a_k$ . Còn $b_{k+1}=Aa_1a_2...a_n$

Ta thấy $b_i-b_j= a_i-a_j |a_i|b_i$ và $b_i-b_j= a_i-a_j |a_j|b_j$ ; $\forall 1\leq i,j\leq n$

và $b_i-b_j$ ko là ước của $b_k$ nào nữa

Riêng $b_i-b_{n+1}= a_i |gcd(b_i,b_{n+1});\forall 1\leq i\leq n$ và cũng ko là ước của $b_k$ nào nữa.

Vậy tập $S_{n+1}=\left \{ b_1,b_2,...b_{n+1} \right \}$ thỏa đề và ta có $đpcm$

Dạ anh giải thích rõ hơn chỗ này được không ạ? Vì giả sử là ước của một $b_k$ nào đó thì $a_i|a_k$ Từ chỗ này em vẫn chưa tìm ra vô lí.




#607959 $a-b|a$ và $a-b|b$

Gửi bởi Bui Ba Anh trong 08-01-2016 - 15:17

Chứng minh rằng với mỗi số tự nhiên $n$ lớn hơn $1$, luôn tồn tại một tập $S$ gồm $n$ phần tử sao cho với hai phần tử $a,b$ bất kì trong $S$, $a-b|a$ và $a-b|b$ nhưng $a-b$ không là ước của bất kì số nào trong $S$ khác nữa.




#603355 Đề cử Thành viên ấn tượng năm 2015 của Diễn đàn Toán học

Gửi bởi Bui Ba Anh trong 15-12-2015 - 20:15

1.Tên ứng viên: Zaraki

2.Thành tích-Đóng góp nổi bật: thành viên trong ban đề xướng, ra đề và chấm thi VMEO 2015, dù đã năm 11 và nhiều việc hơn nhưng vẫn thường xuyên tham gia và đóng góp cho diễn đàn. Hoàn toàn xuất phát từ đam mê và tâm huyết với VMF

3. Chú thích:nope




#594818 chọn đội tuyển tỉnh Đaklak 2015-2016

Gửi bởi Bui Ba Anh trong 22-10-2015 - 15:24

 

KÌ THI LẬP ĐỘI TUYỂN DỰ THI QUỐC GIA -NGÀY 1

Câu 3 (5 điểm)

Cho tam giác $ABC$,$M$ là điểm nằm trong của tam giác.Gọi khoảng cách từ $M$ đến cạnh $AB,BC,CA$ lần lượt là $d_c,d_a,d_b$ và khoảng cách từ $M$ đến các đỉnh $A,B,C$ lần lượt là $x,y,z$.Chứng minh rằng

$\frac{x+y+z}{d_a+d_b+d_c}\ge 2$

 

Câu 4 (5 điểm)

Giải hệ phương trình

$\left\{\begin{matrix} 2(x+y+z)-15=0\\2x+4y+7z-2xyz=0 \\\frac{1}{\sqrt{3x}}+\sqrt{10y}+\sqrt[4]{8z}=\frac{22}{3} \end{matrix}\right.$

 

Chắc câu 1 và 2 là khó của đề 

ý tưởng câu 3 và 4

Câu 3: dễ thấy định lý Erdos-mordel

câu 4: VNTST 2001




#594479 một hộp có 40 thẻ được đánh thứ tự từ 1->40 .Hỏi có bao nhiêu cách lấy 3...

Gửi bởi Bui Ba Anh trong 19-10-2015 - 20:51

một hộp có 40 thẻ được đánh thứ tự từ 1->40 .Hỏi có bao nhiêu cách lấy 3 tấm thẻ , trong đó tổng các số ghi trên 3 tấm thẻ chia hết cho 3

Số trên ba tấm thể nhận số dư lần lượt là $(0,0,0);(1,1,1);(2,2,2);(0,1,2)$ khi chia cho $3$ thì chắc chấn ba tấm đó có tổng chia hết cho $3$

có $14$ số chia $3$ dư $1$, $13$ số chia $3$ dư $0$ và $13$ số chia $3$ dư $2$

số cách chọn là $14C_3+13C_3+13C_3+14.13.13=3094$ cách




#594445 $(a^5-a^2+2)(b^5-b^2+2)(c^5-c^2+2)\geq (a+b+c)^3$

Gửi bởi Bui Ba Anh trong 19-10-2015 - 17:51

Cho a,b,c dương. Chứng minh rằng:
$(a^5-a^2+2)(b^5-b^2+2)(c^5-c^2+2)\geq (a+b+c)^3$

Bài này của Mỹ $2004$, hình như là $+3$ chứ không phải $+2$




#592774 tìm số tập $\mathcal{B}$ sao cho $\min...

Gửi bởi Bui Ba Anh trong 08-10-2015 - 20:47

Gọi tập $\mathcal{A}=\left \{ 1,2,...,n \right \}$.Tìm số tập $\mathcal{B}=\left \{ i_1,i_2,...,i_k \right \}$ là tập con khác rỗng của $\mathcal{A}$ sao cho 

$\min\left \{ i_1,i_2,...,i_k \right \}\ge k$.

Cố định $k->k_0,1 \leq k_0 \leq n$

Vì mọi số trong $|B|=k_0$ đều không bé hơn $k_0$ nên nhận giá trị trong $k_0,k_0+1,...,n$

Số tập $B$ là $C_{n-k_0+1}^{k_0}$

Cho $k_0$ chạy từ $1->n$ thì số tập $B$: $\sum_{k=1}^{n}C_{n-k+1}^{k}=\sum_{k=1}^{n}C_{n-[\dfrac{n+1}{2}]+1}^{[\dfrac{n+1}{2}]}$ (coi như $C_a^b=0$ nếu $b>a$)




#592087 Đề thi chọn đội tuyển lần 2 trường THPT chuyên Hưng Yên

Gửi bởi Bui Ba Anh trong 04-10-2015 - 20:16

Thay vì chứng minh $\sum \frac{1}{1-ab}\leq \frac{9}{2}$. Ta chứng minh$\sum \frac{1}{ab-1}\geq \frac{9}{-2}$

 

Với $\sum a^2=1$ , $BCS- Engel$ cho ta:

$$\sum \frac{1}{ab-1}\geq \frac{9}{\sum ab -3}\geq \frac{9}{\sum a^2 -3}=\frac{9}{-2}$$

Vậy ta có đpcm.

mẫu thức âm đâu dùng được BCS đâu bạn